A bald eagle moving with constant acceleration increases its speed from 5,000m / s to 12.00m / s over a distance of 200.0m. How long does it take you to do this?

a)4.849s
b)23.53s
c)28.95s
d)11.76s

Answers

Answer 1

Answer:

B

Step-by-step explanation:


Related Questions

A few years ago, a country consumed 19,400,000 barrels of oil per day. That same year, there were about 119,000,000 households in that country. A barrel of oil is 42 U.S. gallons. Estimate how many gallons each household in the country consumed each day that year by rounding down all three numbers. (Round 19,400,000 barrels to 19,000,000 barrels, 119,000,000 households to 115,000,000 households, and 42 gallons to 40 gallons. Round your answer to two decimal places.)

_________ gal/household/day

Calculate the answer without using estimation and compare the two results. (Round your answer to two decimal places.)
___________gal/household/day

Answers

Following are the calculation to fill in the blank:

For Part a)

Each home in this town consumed

[tex]\to 40 \times \frac{19000000}{115000000}\\\\\to 40 \times \frac{19}{115}\\\\\to 40 \times 0.165\\\\\to 6.6 \ \text{gallons of oil per day}[/tex]

For Part b)

Without completing the sentence,

[tex]\text{Total consumption = 19200000 barrels}\\\\\text{Total number of households = 118000000}[/tex]

As a result, each household consumed.

[tex]\to \frac{19200000}{118000000} \\\\\to \frac{192}{1180} \ (barrels)\\\\[/tex]

In the given scenario, each barrel includes 42 gallons of oil. Therefore, every household consumed [tex]42 \times \frac{192}{1180} = 6.83 \ gallons[/tex] oil each day.

Learn more:

brainly.com/question/14798687

Hence, each household will consume 6.60 gallons per day.

Total Households of that country =115000000 (approx.)

Total barrels used per day =19000000 (approx.)

Given 1 barrel = 40 gallons (approx.)

So total gallons used per day = 19000000 x 40 =760000000

So each household will consume gallons per day = total gallons used per day / Total household of that country

=[tex]=\frac{760000000}{ 11500000}[/tex]

=6.60 gallons/household/day

So each household will consume 6.60 gallons per day.

Learn More: https://brainly.com/question/12272570

Line I and h intersect at what point

Answers

Answer:

please show where the lines are graphed in a picture

Step-by-step explanation:

If f(x) = 2x + 1 and g(x) = x-2 what is the value of f(g(f(3)))? This is so confusing plz help me will mark brainliest A) 1 B) 3 C) 5 D) 7 E) 11

Answers

Answer:

11

Step-by-step explanation:

f(3) = 2*3+1 = 6+1 = 7

Then find g(7)

g(7) = 7-2 = 5

Then find f(5)

f(5) = 2*5 +1 = 10+1 = 11

f(g(f(3))) = 11

Answer:

11

Step-by-step explanation:

f(x)=2x + 1

f(3)= 2.3 + 1

f(3)=7

f(g(f(3))) = f(g(7))

Now, we have to find g(7)

g(x)=x -2

g(7)= 7 -2

g(7)=5

f(g(f(3))) = f(g(7)) = f(5)

now we have to find f(5)

f(x)=2x + 1

f(5)=2.5 + 1

f(5)=11

Hope this helps ^-^

Which equation represents a line parallel to the line whose equations is -2x + 3y =
-4 and passes through the point (1,3)?

Answers

Answer:

2.  y - 3 = 2/3 (x - 1).

Step-by-step explanation:

-2x + 3y = -4

3y = 2x - 4

y = 2/3 x - 4/3 - so the slope is  2/3.

The slope of a line parallel to it is also 2/3.

It also passes through the point (1, 3).

Using the point-slope form of  a line:

y - y1 = m(x - x1) where m = the slope and (x1, y1) is a point on the line, we have:

y - 3 = 2/3 (x - 1)  <--- is the required equation.

Answer:

2

Step-by-step explanation:

parallel lines have same slope with different intercept

y= mx+b

m is going to be same with different b

so the given function is:

-2x+3y=-43y= 2x-4y= 2/3x - 4/3

Given options:

1. y-3= - 2/3(x-1)  ⇒ y= -2/3x +3 + 2/3 ⇒ y= -2/3x +11/3

it has different slope, so is not parallel

2. y-3= 2/3(x-1) ⇒ y= 2/3x+3-2/3 ⇒ y= 2/3x + 7/3

it has same slope, so is parallelit should be passing through point (1,3)3= 2/3+7/3 ⇒ 3=3, yes it does

3. y-3= -2/3(x+1) ⇒ y= - 2/3x +3- 2/3 ⇒ y= -2/3x + 7/3

it has different slope, is not parallel

4. y-3= 2/3(x+1) ⇒ y= 2/3x +3+ 2/3 ⇒ y= 2/3x +11/3

it has same slope, so is parallelit should be passing through point (1,3)3= 2/3+11/3 ⇒ 3≠13/3, no it doesn't

Tiffany cells 2 kinds of homemade tomato sauce

Answers

Answer:

b

Step-by-step explanation:

Let x represent the number of quarts of Tuscan sauce and

y represents the number of quarts of marinara sauce Tiffany makes.

A quart of Tuscan sauce requires 6 tomatoes and 1 cup of oil

x quarts requires 6x tomatoes and 1x cups of oil

A quart of her marinara sauce requires 5 tomatoes and 1.25 cups of oil

y quarts requires 5y tomatoes and 1.25 y cups of oil

She has 45 tomatoes and 10 cups of oil on hand.

So the constraints are

6x+5y≤45

1x+1.25y≤10

x>=0 and y>=0

After drawing a diagram of what's required to make 1 quart of Tuscan and 1 quart of Marinara sauce, you can more clearly choose answer choice (B) 6x + 5y < (or equal to) 45, x + 5/4y < (or equal to) 10, x>(or equal to)0, y>(or equal to)0.

please mark brainliest :)

the answer is B !!! hope it’s correct

PLEASE HELP WILL GIVE BRAINLIEST AND 20 POINTS

solve this system of equations

3y-5x=12
y=1/3x

Answers

Answer:

x=-3

y=-1

(-3,-1)

Answer:

(-3, -1).

Step-by-step explanation:

I have attached the work to your problem.

Please see the attachment below.

I hope this helps!

What is -6 divided by -1.2?

Answers

Answer:

5.

Step-by-step explanation:

This is the same as saying:

-6/(-6/5).

To solve the equation, we multiply by the reciprocal and simplify. In this case, we multiply both sides of the equation by -5/6.

-6 * (-5/6) = 5.

The answer must be positive, as two negatives divided equal a positive.

I hope this helps!

The solution of expression after divide is,

⇒ 5

We have to given that;

⇒ - 6 is divided by - 1.2

Now, We can simplify as,

⇒ - 6 is divided by - 1.2

It can be written as,

⇒ - 6 ÷ - 1.2

⇒ - 6 / - 1.2

⇒ 6 / 1.2

⇒ 60 / 12

⇒ 5

Thus, The solution of expression after divide is,

⇒ 5

Learn more about the divide visit:

https://brainly.com/question/28119824

#SPJ6

Micheal has 1/2 of a yard of fabric to make book covers. Each book cover is made from 1/8 of a yard of fabric. How many book covers can Micheal make?

Answers

Answer:

4

Step-by-step explanation:

[tex]\frac{1}{2}[/tex] ÷ [tex]\frac{1}{8}[/tex] = 4

Solve:
-3(7p + 5) = 27
Helpppp

Answers

Answer:

-2

Step-by-step explanation:

-21p - 15 = 27

-21p = 42

p = -2

Answer:

-2

Step-by-step explanation:

-21p - 15 = 27

-21p = 42

p = -2

evaluate the following expression -7x(7+9)

Answers

Answer:

-112

Step-by-step explanation:

Add

-7(7+9)

Multiply

-7x16

Wallah! You have the answer

-112

[tex]\text{Simplify the expression:}\\\\-7x(7+9)\\\\\text{Use the distributive property}\\\\-49x-63x\\\\\text{Combine like terms}\\\\\boxed{-112x}[/tex]

Becky made 4 quarts of chicken noodle soup in a big pot, then served 112-cup bowls of soup to 6 people. How much soup is left in the pot?

Answers

Answer:

1qt 3c

Step-by-step explanation:

HELP ASAP it says my questions should be at least 20 characters long so what I’m saying right now doesn’t mean anything.

Answers

Answer: See pic above for answer. I got it from Photomath

Plz put brainliest

14. Roger is on a playground swing, and he is swinging back and forth in such a way that the height, h, in feet, of the swing off the ground is given by the equation h=3cos(3π/2t) +5, where t is in seconds. How many seconds elapses between two consecutive times that the swing is at its maximum height?​

Answers

Answer:

The time [tex]t = \frac{3}{2}[/tex]  seconds elapses between two consecutive times that the swing is at its maximum height  'h' = 2

 Step-by-step explanation:

Explanation:-

Step(i):-

Given function [tex]h(t) = 3 cos (\frac{3\pi }{2 t} ) +5[/tex] ....(i)

By using derivative formulas

[tex]\frac{d cosx }{d x} = -sinx[/tex]

[tex]\frac{d x^{n} }{d x} = n x^{n-1}[/tex]

[tex]\frac{d t^{-1} }{d x} = -1 t^{-1-1} = - t^{-2} = \frac{-1}{t^{2} }[/tex]

Step(ii):-

Differentiating equation(i) with respective to 't'

[tex]h^{l} (t) = 3(-sin(\frac{3\pi }{2t})\frac{d}{dt} (\frac{3\pi }{2t } )+0[/tex]  ...(ii)

[tex]h^{l} (t) = 3(-sin(\frac{3\pi }{2t})(\frac{-3\pi }{2t^{2} } )+0[/tex]

Equating zero

[tex]h^{l} (t) = 3(-sin(\frac{3\pi }{2t})(\frac{-3\pi }{2t^{2} } )=0[/tex]

[tex]3(-sin(\frac{3\pi }{2t})(\frac{-3\pi }{2t^{2} } ) = 0[/tex]

on simplification , we get

[tex](sin(\frac{3\pi }{2t}) = 0[/tex]

now we use formulas

sin 0 = 0 and sinπ = 0

General solution

[tex](sin(\frac{3\pi }{2t}) = sin\pi[/tex]

[tex](\frac{3\pi }{2t}) = \pi[/tex]

Cancellation 'π' on both sides, we get

[tex]3 = 2 t[/tex]

Dividing '2' on both sides , we get

[tex]t = \frac{3}{2}[/tex]

Again differentiating with respective to 't' , we get  

[tex]h^{ll} (t) = 3(-cos(\frac{3\pi }{2t})(\frac{-3\pi }{2t^{2} } )+ (-3)(-sin(\frac{3\pi }{2t} )(\frac{6\pi }{2t^{3} }[/tex]

Put t= 3/2 and simplification

[tex]h^{ll} (t) < 0[/tex]

The maximum height

                   [tex]h(t) = 3 cos (\frac{3\pi }{2 t} ) +5[/tex]

               [tex]h(\frac{3}{2} ) = 3 cos (\frac{3\pi }{2(\frac{3}{2} )} )+5[/tex]

              [tex]h(\frac{3}{2} ) = 3 cos (\pi )+5 = -3+5 =2[/tex]

[tex]t = \frac{3}{2}[/tex]  seconds elapses between two consecutive times that the swing is at its maximum height  'h' = 2

Conclusion:-

The time [tex]t = \frac{3}{2}[/tex]  seconds elapses between two consecutive times that the swing is at its maximum height  'h' = 2

 

 

What is the midpoint of the segment below? (2,3)(-3,-2)

Answers

Answer:

(-0.5, 0.5)

Step-by-step explanation:

If those two points are endpoints, just use the midpoint formula.

(-0.5, 0.5)

Answer:

(-1/2,1/2)

Step-by-step explanation:

To find the midpoint, add the x coordinates and divide by 2

(2+-3)/2 = -1/2

add the y coordinates and divide by 2

(3+-2)/2 = 1/2

The midpoint is (-1/2,1/2)

00:00
Brice is finding the sum of 468 and 241 by breaking it into smaller problems
He uses place value and finds the sums of the hundreds, tens, and ones.
What is the sum of the tens? Enter your answer in the box.
1​

Answers

468+241=709 so 0 is in the tenth place because 6+4= 10 and one gets carried over to the 4+2

I need help with is question ASAP please

Answers

The y-intercept looks like it's 0 and the slop looks like 6/2 or 3 or maybe 7/2

What is the equation in slope-intercept form of a line with slope of 5 and y-intercept of 3?

Answers

y=5x+3

Because the starting value(y intercept is 3) and the slope(x) is 5

Is the relation a function?
{(-6, -1), (5,-1), (0, -1), (-2, -1), (3, -1)}

Answers

Answer:

yes it is a function

Step-by-step explanation:

For that no two x values are the same with different y values

A. 256
B. 265
C. 297
D. 279​

Answers

Answer:

A.

Step-by-step explanation:

So calculate one of the triangles.

[tex]\frac{12*8}{2} =48[/tex]

48 * 4 = 192

192 + 64 = 256

please help as soon as possible
I WILL MARK YOU AS BRAINLIEST ​

Answers

Answer:

Last choice

Step-by-step explanation: Subtract 11 and then divide by 3. A would be greater than -2 and last choice shows that


[tex]( {2}^{ - 1} + {3}^{ - 1} )^{2} [/tex]
solve.

will give the brainliest​

Answers

Answer:

25/36

Step-by-step explanation:

(2^-1 + 3^-1)^2

(1/2 + 1/3)^2

(5/6)^2 = 25/36

25/36 or .694

2^-1= 1/2
3^-1=1/3

Add the together
5/6 square it and you get 25/36

The area of the base of the cone is 64π square millimeters, and the area of the lateral surface is 112π square millimeters. Find the radius r and slant height ℓ of the cone.

Answers

Answer:

Radius = 8 mm

Slant Height = 14 mm

Step-by-step explanation:

Base of a cone is a circle.

Area of circle is given as [tex]A=\pi r^{2}[/tex]

Where [tex]r[/tex] is the radius.

Given that

[tex]A = 64 \pi\ mm^2\\\Rightarrow \pi r^{2} = 64\pi \\\Rightarrow r^{2} = 64\\\Rightarrow r = 8\ mm[/tex]

Hence, radius is 8 mm.

Formula for Lateral Surface Area of a cone:

[tex]LSA = \pi rl[/tex]

Where [tex]r[/tex] is the radius and

[tex]l[/tex] is the slant height of cone

Given that [tex]LSA = 112 \pi\ mm^2[/tex]

[tex]\Rightarrow 112\pi = \pi \times 8 \times l\\\Rightarrow 8 \times l = 112\\\Rightarrow l = 14\ mm[/tex]

Hence, slant height is 14 mm

BEST ANSWER GETS TO CHOOSE BRAINLESST OR FOLLOW!​

Answers

Answer:13 is>

14 is =

15 is >

16 is<

17 is<

18 is =

Step-by-step explanation:

I just know plz mark brainliest



A new car is purchased for 23400 dollars. The value of the car depreciates at 11.5% per year. To the nearest year, how long will it be until the value of the car is 12700 dollars?

Answers

Answer: 5 years

Step-by-step explanation:

Using same formula as for Compound Interest:

12700 = 23400 (1-.115)^t

12700 = 23400 x 0.885^t

0.885^t = 127/234

Convert decimal into fraction:

177/200^t = 127/234

Take logarithm of both sides of thr equation:

t = log 177/200 (127/234)

t = 5.00242

Find the solution(s) to x^2- 14x + 49 = 0.
O A. x=-2 and x = 7
B. x= -1 and x = 14
C. x= 7 only
D. x = 7 and x = -7

Answers

Answer:

[tex] \boxed{C. \: x = 7 \: only} [/tex]

Step-by-step explanation:

[tex] = > {x}^{2} - 14x + 49 = 0 \\ \\ = > {x}^{2} - (7 + 7)x + 49 = 0 \\ \\ = > {x}^{2} - 7x - 7x + 49 = 0 \\ \\ = > x(x - 7) - 7(x - 7) = 0 \\ \\ = > (x - 7)(x - 7) = 0 \\ \\ = > {(x - 7)}^{2} = 0 \\ \\ = > x - 7 = 0 \\ \\ = > x = 7[/tex]

Pls help me on this................

Answers

Answer:

Step-by-step explanation:

Helppppp meeeee answer thissssssss

Answers

Answer:

(-1.025)^3 x (-1.025)^2=(- 1.025)^5

(-y)^3 x (-y)^2 x (-y)= y^6

5= odd, - stays

6= even, - becomes +

Answer:

5) (-1.025)^3* (-1.025)^2 = (-1.025)^5 = (-41/40)^5

10) -y^3 * -y^2 * -y

Multiplying an odd number of negative terms makes the product negative

= - y^3 * y^2 * y

= - y^6

Arrange from smallest to largest
0.01, 1.01, 10.01, 1.001

Answers

Answer:

0.01 --> 1.001 ---> 1.01 ---> 10.01

The numbers in the ascending order:

0.01, 1.01, 1.001, 10.01.

What is ascending order?

Putting numbers in ascending order simply means to do it from smallest to largest.

Given:

0.01, 1.01, 10.01, 1.001.

The numbers from smallest to largest:

That means, in the ascending order.

So,

0.01, 1.01, 1.001, 10.01.

Therefore, 0.01, 1.01, 1.001, 10.01 are in the required order.

To learn more about the ascending order;

https://brainly.com/question/20681445

#SPJ2

If g(x) = -4x + 5, find g(2x-1).

Answers

Answer:

- 8x + 9

Step-by-step explanation:

To evaluate g(2x - 1) substitute x = 2x - 1 into g(x), that is

g(2x - 1)

= - 4(2x - 1) + 5 ← distribute parenthesis and simplify

= - 8x + 4 + 5

= - 8x + 9

Answer:

= - 8x + 9

Step-by-step explanation:

= - 8x + 4 + 5

= - 8x + 9

Find the value of x in the triangle pair below

Answers

Answer:

x=16.97056274... or 12[tex]\sqrt{2}[/tex]

Step-by-step explanation:

the hypotenuse of a 45, 45, 90 triangle is the length of the leg times the square root of 2

Other Questions
The whole class laughed loudly at the story.simple subject: simple predicate: IF 2+2=4 AND 5+5 IS 10 __________ A rose has a phenotype ratio of 3:1 where red roses are dominant over white roses. Out of a total of 4 roses, how many of the roses are red? 1 2 3 4 Takeisha is placing placemats around a table. Each placemat is 14 inches long and 10 inches wide. The rectangular table is 42 inches long and 48 inches wide. What is the greatest number of placemats that Takeisha can place around the table without overlapping? You are thinking about the things that can go wrong on your trip home over the Thanksgiving break. You have booked a flight with US-Scareways. You know that in 33 percent of the cases the company has canceled the flight you were on. Should such a thing occur, there would be no other air travel option home for you. As a backup, your friend Walter has offered you a ride back. However, you know that Walter only has a seat in his car for you with 88 percent probability. What is the probability of you making it home for the holidays? An article presents a study that investigated the effect of varying the type of fertilizer on the height of certain Mediterranean woody tree species. In one experiment, three samples, each consisting of ten trees, were grown with three different fertilizers. One, the control group, was grown with a standard fertilizer. Another was grown with a fertilizer containing only half the nutrients of the standard fertilizer. The third was grown with the standard fertilizer to which a commercial slow-release fertilizer had been added. Following are the heights of the trees after one year. Height Fertilizer Control 17.9 12.2 14.9 13.8 26.1 15.4 20.3 16.9 20.8 14.8 Deficient 7.5 74 13.8 116 115 17 132 129 176 9.5 Slow-releasel 19.8 20.3 16.1 179 124 12.5 174 19.9 27.3 14.4 a) If you want to construct a hypothesis to test for heights of the trees, state your null and alternative hypothesis. b) Is the experiment a factorial balanced CR design? c) What is the degree of freedom of treatment sum of square and error sum of square? If SSTrt and SSE is given: SSTrt=192.374 SSE=439.389, what is the value of your test statistics? d) Compute the SST? e) We get a p-value of 0.007 and we have significance level 0.05 for this case, state your conclusion of the test. After Clara ran 8 times around a square field , she covered a distance of 288km. Calculate the area of the field. What number should be added to both sides of the equation to complete the square? x2 + 3x = 6 (StartFraction 3 Over 2 EndFraction) squared 3 62 Can someone help me I was doing good and then I got stuck on this question what I cant figure out is how you get the voltage when you only have the watt and resistance . A 60.0 Watt lightbulb has an operating resistance of 220 .Calculate the operating voltage of this lightbulb. Which of the following descriptions is correctly paired with its viral cycle?A. Lytic cycle: Viral DNA is integrated into the host's DNA for replication.B. Lytic cycle: Integrated viral DNA can cause the host cell to become more pathogenic.C. Lysogenic cycle: Infection remains hidden until many years later when lysis and cell death occur.D. Lysogenic cycle: Invading viruses direct the cell to produce enzymes that break down the cell wall. Since all living things on Earth share the same universal code in DNA and RNA molecules, which of the following is most likely true ? Write the polynomial in standard form.- 4 + 3x - 522 The goal of this exercise is to determine whether the following system specifications are consistent: If the file system is not locked, then new messages will be queued. If the file system is not locked, then the system is functioning normally, and conversely. If new messages are not queued, then they will be sent to the message buffer. If the file system is not locked, then new messages will be sent to the message buffer. New messages will not be sent to the message buffer. The trial balance columns of the worksheet for Flint at March 31, 2019, are as follows. Flint Worksheet For the Month Ended March 31, 2019 Trial Balance Account Titles Dr. Cr. Cash 4,900 Accounts Receivable 3,400 Supplies 1,800 Equipment 11,176 Accumulated DepreciationEquipment 1,270 Accounts Payable 2,500 Unearned Service Revenue 700 Common Stock 10,266 Retained Earnings 2,700 Dividends 1,200 Service Revenue 6,700 Salaries and Wages Expense 1,300 Miscellaneous Expense 360 24,136 24,136 Other data: 1. A physical count reveals only $700 of roofing supplies on hand. 2. Depreciation for March is $254. 3. Unearned service revenue amounted to $200 at March 31. 4. Accrued salaries are $600.A. Enter the trial balance on a worksheet and complete the worksheet.B. Prepare an income statement for the month of March.C. Prepare a retained earnings statement for the month of MarchD. Prepare a classified balance sheet at March 31.E. Journalize the adjusting entries from the adjustments columns of the worksheet.F. Journalize the closing entries from the financial statement columns of the worksheet. Shipments of product X from a plant to a wholesaler are made in lots of 600 units. The wholesaler's average demand for product X is 150 units per week. Lead time from plant to wholesaler is 4 weeks. The wholesaler pays for the shipments when they leave the plant. What is the total of the wholesaler's cycle inventory and pipeline inventory? An object has a kinetic energy of 35j and a mass of 24j kg,how fast is the object moving? Why would a nation bring a case before the World Trade Organization? A. to create new barriers to trade B. to start businesses in developing countries C. to settle a trade dispute with another country Adolf Hitler took control of the Nazi Party in1918192119291932Please help me !! Solve and graph. Thank you so much! The computer workstation furniture manufacturing that Santana Rey started in January is progressing well. As of the end of June, Business Solutions's job cost sheets show the following total costs accumulated on three furniture jobsJob 602 Job 603 Job 604Direct Materials $1,700 $3,100 $2,600Direct labor 1,000 1,340 1,800Overhead 500 670 900Job 602 was started in production in May, and these costs were assigned to it in May: direct materials, $500; direct labor, $250; and overhead, $125. Jobs 603 and 604 were started in June. Overhead cost is applied with a predetermined rate based on direct labor costs. Jobs 602 and 603 are finished in June, and Job 604 is expected to be finished in July. No raw materials are used indirectly in June. (Assume this companys predetermined overhead rate did not change over these months).1. What predetermined overhead rate is used in June?2. How much cost is transferred to finished goods inventory in June?